Você está na página 1de 47

FINANCIAL PLANNING ACADEMY

RETIREMENT PLANNING

Part 1
1 What is the minimum number of employees in an establishment for it to come under the
preview of Payment of Gratuity Act?
(a) 20 and above
(b) Above 20
(c) 10 and above
(d) Above 15

2 What is the maximum amount of gratuity payable as per the Act?


(a) Rs 300000
(b) Rs 350000
(c) Rs 500000
(d) Rs 510000

3 How much amount of gratuity is payable for each year of completed service to the
monthly rated employees?
(a) 15 days’ wages
(b) Half month’s wages
(c) 20 days’ wages
(d) 30 days’ wages

4 Wages for the purpose of gratuity payment as per the Act means
(a) Basic pay
(b) Basic pay and dearness allowance
(c) Basic pay, dearness allowance and city compensatory allowance
(d) Pay including all allowances

5 Can an employer pay gratuity higher than that prescribed under the Act?
(a) Yes, but within the prescribed ceiling
(b) No
(c) Yes, but always at the prescribed rate for each year of completed service
(d) Yes, at the same or a higher rate and with or without the prescribed ceiling

6 What is the maximum Tax free gratuity in case of Government employees?


(a) Full amount of gratuity
(b) Rs.500000
(c) Rs 250000
(d) Rs 350000

Financial Planning Academy 1


7 For monthly rated employees, to determine the amount of wages for one day for
calculation of gratuity amount, the month is reckoned of
(a) 30 days
(b) 26 days
(c) 31 days
(d) 365/12 days

8 Mr. Gupta joined a private company on 01.10.1965.He retired from the services on
31.05.2004 at superannuation age of 60 years when his monthly emoluments were as
under.
Basic salary: Rs 8000
Dearness Allowance Rs 2000
City compensatory allowance Rs 1000
HRA Rs 5000

During the period of service he had availed without pay leave of 3 months. The company
pays gratuity as per the provisions of the payment of Gratuity Act,1972. Mr. Gupta is
entitled to get the gratuity amount of:
(a) Rs 350000
(b) Rs 230231
(c) Rs 225000
(d) Rs 328846

9 An employee joined a woollen garments concern in 1993. After working all the years
as a seasonal employee up to the year 2004, he retires with the following monthly salary:
Basic salary: Rs 2000
Dearness allowance Rs 1000
HRA Rs 500

How much amount of gratuity is payable to him?


(a) Rs 11308
(b) Rs 9692
(c) Rs 8884
(d) Rs 20769
(Hint: for seasonal workers gratuity amt=7/26 basis)

Financial Planning Academy 2


10 Mr. Pathak joined a company on 15th April 2004 and died in an accident on 30th
October 2004.The company pays gratuity as per the payment of Gratuity Act. His
terminal monthly wages was:
Basic Pay Rs 4500
Dearness Allowance Rs 3000
City Compensatory Allowance Rs 500
HRA @20% of the basic Pay

His nominees will get as gratuity an amount of


(a) Rs 4326
(b) Rs 4500
(c) Rs 4615
(d) Rs 4205
(There is no condition of completion of 5 Years in case of death)

11 Is pension a statutory benefit in India and if so, for which segment of employees?
(a) All employees working in any sector
(b) All government employees
(c) All employees covered under the EPF and Miscellaneous Provisions Act, 1952
(d) All public sector employees

12 What is the maximum amount of pension that can be commuted as per Rule 90 of the
Income Tax Rules?
(a) 40% of the pension
(b) One half of the pension if the employee does not get gratuity otherwise one third
of the pension.
(c) One third of the pension
(d) One half of the pension

13 The PPF account can be opened in


(a) SBI or its subsidiaries
(b) Head Post Office or any selection grade sub PO
(c) Any Nationalized Bank
(d) Any one of the above

14 For PPF a/c an individual can open


(a) An account in his own name and an additional account in the name of a minor of
whom he is the guardian.
(b) An account in the name of brother or sister
(c) An account in the name of father or mother
(d) None of these

Financial Planning Academy 3


15 The minimum and maximum amount can be deposited in a PPF account in a year are
(a) 500,70000
(b) 100,100000
(c) 500,100000
(d) 250,700000

16 Investment in a PPF qualify for Tax benefit under section


(a) Sec 88
(b) Sec 80 L
(c) Sec 80 C

17 How many PPF accounts can be opened in one name?


(a) One
(b) Two
(c) Three
(d) Four

18 The PPF account as initially meant for 15 years and then the subscriber can exercise
his option for extension for a block of :
(a) 5 years
(b) 10 years
(c) 15 years
(d) 2 years

19 In which year can the subscriber to a PPF account take the first loan from the opening
of the account?
(a) Third year
(b) Fifth year
(c) Second year
(d) Seventh year

20 The PPF account was opened in the year 1999-2000, when can the first withdrawal
be made?
(a) After 31-3-2005
(b) After 31-3-2004
(c) After 31-3-2002
(d) After 31-3-2003
(HINT: WITHDRAWAL CAN BE MADE IN THE SEVENTH YEAR AND YEAR STARTS
FROM IST APRIL AND ENDS ON 31ST MARCH)

Financial Planning Academy 4


21 The amount standing to the credit of the subscriber in a PPF account is:
(a) Exempt from the Wealth Tax
(b) Free from attachment by a court in respect of any debt or liability incurred by the
subscriber
(c) Subject to attachment under the orders of the Income Tax authorities
(d) All of the above

22 The interest credited to the PPF is


(a) Exempt from the tax
(b) Allowed benefit of sec 80 C
(c) Fully taxable
(d) None of these

23 Which of the following is false?


(a) An NRI can open a PPF account
(b) The parents can open PPF account on behalf of their minor children
(c) A discontinued account cannot be closed before the date of maturity
(d) A resident who had opened a PPF account can continue his account till its maturity on
becoming an NRI.

24 Which of the following statements are true?


A subscriber having a discontinued PPF account cannot
(a) Open a new PPF account
(b) Raise loan
(c) Take withdrawals
(d) All of the above

25 From which date the “Employees Pension Scheme” become effective?


(a) 1st January, 1995
(b) 16th November, 1995
(c) 1st January, 1996
(d) 16th Feb, 1995

26 The Employees Provident Funds and Miscellaneous Provisions Act, 1952 extends to
(a) Whole of India
(b) A few specified states of India
(c) All States excluding Union Territories
(d) Whole of India except the State of Jammu and Kashmir.

Financial Planning Academy 5


27 Illness, a non-refundable advance from the PF may be allowed to a member in cases
of:
(a) Hospitalisation lasting for one month or more, or
(b) Major surgical operation in a hospital, or
(c) Suffering from TB, Leprosy, paralysis, cancer, mental derangement or heart ailment
(d) Any of these

28 The amount advanced for illness shall not exceed the member’s own contribution with
interest in the fund or
(a) 24 months’ salary (Basic+DA) of the member
(b) 6 months’ salary (Basic+DA) of the member
(c) 36 months’ salary (Basic+DA) of the member
(d) 12 months’ salary (Basic+DA) of the member

29 To avail advance from PF for the member’s own marriage, the marriage of his or her
daughter, son, sister or brother or for the post matriculation education of his or her son or
daughter, the member should have completed:
(a) 2 years’ membership
(b) 10 years’ membership
(c) 5 years’ membership
(d) 7 years’ membership

30 An employee having an average balance of Rs.90000 in the PF during the last 12


months dies in an accident, what amount of insurance cover is payable to his nominee
under the EDLI Scheme of the PF authorities?
(a) Rs. 90000
(b) Rs 60000
(c) Rs 41250
(d) Rs 48750
(Hint: 35000+ {(90000-35000)/4} subject to maximum of Rs 60000)

31 From which date the Employees Pension Scheme become effective?


(a) 1st January 1995
(b) 16th November 1995
(c) 1st January 1996
(d) 16th January,1995

32 The “Eligible service” for the purpose of the EPS is


(a) The aggregate of the “actual service” and the “ past service”
(b) The actual service only
(c) The past service only
(d) None of these

Financial Planning Academy 6


33 A member shall be entitled to pension if has rendered minimum eligible service of
(a) 5 years
(b) 10 years
(c) 20 years
(d) 33 years

34 “X” promises to pay “Y” A Sum of Rs.30000 at the end of 3 years and another
Rs.50000 at the end of 5 years. What amount should be accepted now in lieu of the above
two payments if interest rate is 6%?
(a) Rs 60000
(b) Rs 62549
(c) Rs 63000
(d) Rs 62995
(HINT: COMPUTE PV UNDER BOTH THE OPTIONS AND ADD THE TWO)

35 An employee aged 35 Years invested Rs. 10000 in a saving instrument. The interest
during the first 5 years is 8% p.a. and thereafter 6% p.a. What amount would he get on
retirement at the age of 58 years?
(a) Rs.41939.61
(b) Rs 45000
(c) Rs 42353.50
(d) Rs 42161.90

36 Calculate the present value of an immediate annual annuity payable for 20 years
certain at the rate of Rs 15000 per annum, the first installment being due at the end of one
year. Assume rate of interest as 7% p.a.
(a) Rs 161250.21
(b) Rs 158910.21
(c) Rs 159682.50
(d) Rs 160205.62

37 Which yields the higher rate of interest for every Rs 1000 in a bank or an NSC giving
the following maturity values respectively?
(a) Rs 1629 after 5 years in a bank FD
(b) Rs 1901 after 6 years in NSC
(HINT: COMPUTE THE RATE OF RETURN I.E I IN CASE OF FD IT IS 10.25% AND NSC
GIVES 11.3%)

Financial Planning Academy 7


38 A sum of Rs 50000 is invested at a rate of 5% p.a. After 7 years the rate of interest
was changed to 5% p.a. convertible half yearly. After a further period of 3 years, the rate
was again changed to 6% p.a. convertible quarterly. Find the accumulated amount at the
end of 15 years from commencement?
(a) 109890.15
(b) 109868.35
(c) 109965.23
(d) 109986.23
(First 7 years 5%p.a.,next 3 years 2.5%per half year, next 5 years 1.50%per quarter)

39 Mr. Gupta deposited a sum of Rs. 2500000 in an annuity certain plan for 25 years
providing yield of 6% p.a. What would be the amount of annuity if he wishes to receive
annual annuity payments in arrears, in advance (annuity due).
(a) 195566.12,184496.34
(b) 195656.15,185549.36
(c) 195869.32,185096.34
(d) 195686.65,184496.34

40 Mr. khanna invested Rs 500000 in a plan allowing withdrawal of Rs 50000 at the end
of each year giving return of 8% p.a.. How long the money will last in the said plan?
(a) 21.91 years
(b) 20.91 years
(c) 18.95 years
(d) 19.25 years

41 Mr Ram aged 45 saves at 9%p.a. Rs. 200000 at the beginning of the year for the first 8
years and then stops saving on account of certain financial problems. On retirement at the
age of 65 years, he intends to keep aside a sum of Rs 500000 out of the accumulated
amount of the above savings as liquid money for emergencies and to invest the balance
amount at 6% p.a. providing withdrawal of a fixed amount at the end of every year for 20
years. Find the amount of annual withdrawal?
(a) 545900.95
(b) 545970.95
(c) 546982.25
(d) 547856.75
(HINT: HE SAVES FOR 8 YEARS AND THEN THE ACCUMULATED AMOUNT GROWS
FOR ANOTHER 12 YEARS WHICH HE INVESTS IN ANNUITY PROVIDING A FIXED
SUM FOR 20 YEARS)

42 Find the present value of an immediate annuity of Rs 1000 payable quarterly (in
arrears) for 9 years at the rate of interest of 9% convertible quarterly.
(a) 25494.67
(b) 24598.66
(c) 24494.66
(d) 25652.52

Financial Planning Academy 8


43 Raghav aged 42 years working in a manufacturing company wants “nest eggs”
amounting to Rs 5000000 at his retirement age of 62 years. How much should he save in
the end of every year until retirement? The available interest rate is 8%.
(a) 109300
(b) 109261
(c) 109256
(d) 109345

44 In the above case if Raghav wants to saves at the end of the year, how much amount
he will save at the beginning of every year to reach to the target of Rs 5000000?
(a) 101167
(b) 101562
(c) 101325
(d) 101150

45 Which government authority accords approval to a scheme providing pensionary


benefits to the employees?
(a) Ministry of Finance
(b) Ministry of Labour
(c) PF Authorities
(d) Income Tax department.

46 Under an educational loan, a person receives four annual installments of Rs. 10000
each, the first payment being made at the present moment. The loan is to be repaid in
lump sum at the end of 10 years from now along with 6% interest p.a. Calculate the
amount repayable.
(a) Rs. 65778.03
(b) Rs. 65889.05
(c) Rs.68559.07
(d) Rs64885.23
( FIRST CALCULATE PV OF THE LOAN AMOUNT(ANNUITY DUE) AND THEN
CALCULATE FV)

47 An employee aged 30 is currently spending Rs 1 lakh per annum. If the rate of


inflation is 5% and he wishes to maintain the same the present standard of living through
out his service , what amount would he be requiring to spend during the first year of his
exit from service if the retirement age is 60 years
(a) Rs 432500.00
(b) Rs 433150.00
(c) Rs 432194.00
(d) Rs 432150.00

Financial Planning Academy 9


48. A has invested Rs.1000 in a savings instrument maturing after 15 years. On its
maturity, he receives a sum of Rs 1750. What rate of interest is realized in this
transaction?
(a) 3.92%
(b) 3.80%
(c) 3.56%
(d) 3.95%

49 Suman aged 30 years is working in an MNC and wishes to set aside some fixed
amount at the beginning of each year towards retirement planning. He is currently
spending Rs 240000 pa and wishes to raise his standard of living by 2% per year until his
retirement at age 55. The average rate of inflation is expected to be 3% all these years. If
he wants to maintain 90% of his standard of living that he would be enjoying on
retirement assuming no provision for inflation thereafter, then what is the annual amount
of income he should manage for after retirement?
(a) Rs 731452.50
(b) Rs 732562.05
(c) Rs 732482.50
(d) Rs 735265.23
(HINT: FIRST CALCULATE THE AMOUNT REQUIRED AT RETIREMENT TO
MAINTAIN SAME STANDARD OF LIVING (2% RISE IN STANDARD OF LIVING
AND 3%INFLATION RATE)) AND THEN CALCULATE 90%OF THE AMOUNT.

50 In the above case, if Suman’s life expectancy on retirement is 25 years and he wants
the same annual income as envisaged in question number 49 above for all the years in
advance after retirement, what accumulated amount should he have on retirement for
such an arrangement ? Assume his investment on retirement would provide a yield of 6%.
(a) Rs 9811562
(b) Rs 9911476
(c) Rs 9758963
(d) Rs 9912458
(HINT: ANNUAL INSTALMENT: RS 731452.50,N=25(ANNUITY DUE),R=6%
COMPUTE:PV)

51 Find Present Value of an annuity certain that pays 50000 at the end of the year for next
10 years, 70000 for next 10 years and 80000 for the last 10 years. Assume rate of interest
to be 6% pa for 20 years and 5% pa for the last 10 years.
(a) 848306
(b) 845869.02
(c) 849568.23
(d) 848453.65

Financial Planning Academy 10


52 Radhika has an option of receiving Rs 100000 now or instalment of Rs 12000 at the
end of the year for next 10 years. Assume rate of interest to be 4% pa, which option is
more profitable?
(a) Cash option
(b) Installment option
(c) Both are same
(d) None of these
(HINT: CALCULATE PV FOR BOTH THE OPTIONS AND COMPARE)

53 Mr. Rajat aged 30 years is working as an architect with an annual income of Rs


600000. He expects that after meeting his annual living expenses, he will be able to save
Rs 150000 every year at the end of the year. Assume rate of interest to be 6%.What
would be the amount of his accumulated savings if he retires at the age of 60 years?
(a) Rs 11568925
(b) Rs 11858727
(c) Rs 11869569
(d) Rs 11859653

54 In the above question what would be the accumulated amount of his saving if he
prepones his retirement to the age of 55 years?
(a) 8229676.80
(b) 8256252.40
(c) 8230576.58
(d) 8229152.32

55 If Mr. Rajat’s accumulated savings are invested at the rate of 7.5% pa on his
retirement and after retirement and the impact of inflation is 5%,how long the money will
last if he retires at the age of 60 and he spends Rs 650000 pa.
(a) 24.21
(b) 35.85
(c) 37.56
(d) 38.56
(HINT: REAL RATE OF RETURN IS 1+NOMINAL RATE/1+INFLATION-1*100=2.38%
WHICH WILL BE USED AFTER RETIREMENT TO CALCULATE NUMBER OF YEARS)

56 Mr. Rajan who is aged 42 years has got a contractual assignment in UAE for a period
of 15 years. He has been on his assignment for the past 4 years and has already saved Rs
7 lakh every year. He now plans to save Rs 10 lakhs for the balance period of his
assignment abroad and then come back to India and live a retired life. If his saving earn
an interest of 7%pa during the accumulated stage, how much accumulated will he have
when he returns to India?
(a) Rs 223.25 lakhs
(b) Rs 225.00 lakhs
(c) Rs 222.3 lakhs
(d) Rs 223.85 lakhs

Financial Planning Academy 11


57 In the above question if his life expectancy is 85 years and the accumulated savings
earn a return of 8%pa ,how much money can he spend per annum to sustain himself for
the balance period of his estimated life span?(calculate on annuity due basis)
(a) 180552.15
(b) 180772.16
(c) 181005.15
(d) 181992.35

58 If the estimated annual expenses for his family are Rs 360000 pa and the rate of
inflation is 6%,then how long will his accumulated savings last?
(a) 35 years
(b) 40 years
(c) 45 years
(d) last forever
(HINT: THE INTEREST EARNED ON INVESTMENT IS MORE THAN ANNUAL LIVING
EXPENSES)

59 If his contract is extended for a further period of 5 years and he is in a position to save
RS 12 lakhs per year during this extended period of stay, how much will he accumulate
by the time he returns to India?(assume: hi s savings will earn a return of 7% pa
throughout the period of total stay abroad.)
(a) 350.56 lakhs
(b) 398.25 lakhs
(c) 382.13 lakhs
(d) 385.16 lakhs

60 Under the changer circumstances as narrated in q.no-59,how much money per annum
is available for household expenses during his balance life span if the inflation is ignored
and his savings earn an interest of 6% during the disbursement period.
(a) 27.288 lakhs
(b) 29.300 lakhs
(c) 28.562 lakhs
(d) 27.562 lakhs

61 A 30 year old employee is currently earning an annual salary of Rs 300000. He has


started saving 10% of his salary at the end of each year in a savings plan yielding 6%
interest pa. His salary increases by 5%pa.What accumulated amount would he be having
on his retirement at age 60?
(a) Rs. 4265640
(b) Rs 4264650
(c) Rs 4234560
(d) Rs 4265890
( This is growing annuity: salary is also increasing every year at constant rate)
Formula for growing annuity is =pmt{(1+r)-(1+g)}/{(1+r)-(1+g)}and (1+r) and (1+g) in
the numerator have raise to power”n”Pmt here is 30000 i.e. 10% of salary in the first year
and n is 30

Financial Planning Academy 12


62 In the above question if he intends to pre-pone his retirement to the age of 55 years
and wants to have the same amount of accumulated savings as at the age of 60 years,
what percentage of his salary should he start saving to achieve his goal?
(a) 15.25%
(b) 15.30%
(c) 15.70%
(d) 16.00%
(FV is Rs 4264650 and with the same formula as given in Q No 61 we have to calculate
Pmt which comes as Rs 47096.14. To calculate saving as % of current salary
47096.14/300000*100=15.7%

Attempt question 63 to 66 on the base of this information


63 A 33 year old employee plans to save 12% of his salary at the end of every year until
his retirement at the age of 58 years. He is currently getting an annual salary of Rs
360000 which is expected to increase 8% every year. If his savings earn 7.5% interest pa,
how much accumulated money will he have on his retirement?
(a) 64.80 lakhs
(b) 65.00 lakhs
(c) 63.89 lakhs
(d) 62.90 lakhs

64 He deposited this amount (as calculated in the above question) in an annuity certain
due for a period of 20 years providing a yield of 5% pa. How much money pa will he get
under this annuity plan? ( BGN mode)
(a) 4.50 lakhs
(b) 4.85 lakhs
(c) 4.95 lakhss
(d) 5.01 lakhs

65 If he seeks an early retirement at the age of 50 years, what percentage of his salary
should he save every year to accumulate the same amount as he would have accumulated
on his normal retirement age?
(a) 32.25%
(b) 35.00%
(c) 32.14%
(d) 33.65%

Financial Planning Academy 13


66 In the above case , he decides to purchase an immediate annuity certain for a period of
30 years giving a yield of 6% pa convertible monthly. How much monthly payment under
this annuity plan will he get?
(a) Rs. 38,858
(b) Rs 38,658
(c) Rs 39,500
(d) Rs 38,950

67 An employee is covered under an unrecognized Provident Fund. He is aged 40 and is


contributing 12% of his salary at the end of every month to this fund. His employer is
also depositing a matching amount in the fund as a social security benefit. The employee
is currently getting a monthly salary of Rs 10000. Assuming that his salary remains static
through out the period of his service until his retirement at the age of 60 years and he
earns 9% return on the investment.
(a) 1603895
(b) 1602928
(c) 1605469
(d) 1603256

68 A good retirement benefit scheme should satisfy the following need of a person:
(a) Need for a regular periodical income after retirement
(b) Need for lump sum cash payment on retirement
(c) Need for reimbursement of expenses connected with medical emergencies
(d) All of the above

69 Gratuity benefit payable by an employer as per the provisions of the Payment Of


Gratuity Act , 1972 is under
(a) Defined Contribution retirement benefit
(b) Defined benefit type of a retirement benefit
(c) Either of the two types of benefit plans
(d) Neither of the two

70 Contributions under a defined benefit plan


(a) Will be fixed in relation to the wages earned
(b) Will be determined based on the benefits assured
(c) Will be fixed as per the designation
(d) Will be decided by the members of the benefit scheme

71 Which of the following is not a defined benefit retirement plan?


(a) Gratuity benefit payable as per the Act
(b) Superannuation scheme as applicable to the Government Servants
(c) Gratuity benefit payable to the Public Sector Banks’ employees as per the agreement
between the employees and the Indian Banks Association
(d) Employees Provident Scheme

Financial Planning Academy 14


72 From the viewpoint of the employer, a defined contribution retirement benefit plan is
easier to operate because
(a) His contributions will be lesser than the defined benefit plans
(b) He does not have to contribute anything to the scheme at all
(c) Since there is no need for valuation the employer’s expense is less
(d) The employer does not have to submit any returns to any authority for approval

73 In a defined contribution retirement plan an change in the rate of interest earned by the
funds would affect
(a) The benefit payable to the employees and not the employer
(b) The contribution payable by the employer and not the benefits payable to employees
(c) Both the employees ‘ benefit and the employers’ contribution
(d) None of these

74 Carrying the retirement benefit plan from one employer to another when an employee
leaves the services of one employer and joins another is easy if the retirement plan is
under
(a) Defined benefit plan
(b) Defined contribution plan
(c) The type of plan makes no difference
(d) None of these

75 Gratuity is payable to all employees who leave the services of an employer(other than
by death or incapacity)after putting in a minimum service of
(a) 15 years
(b) 10 years
(c) 5 years
(d) 12 years

76 The minimum service required for entitlement to payment of Gratuity under the Act,
in case of an employee leaving service by death or incapacitation is
(a) 2 years
(b) 3 years
(c) 5 years
(d) No minimum service is required

77 For calculation of liability of payment of Gratuity to an employee on leaving service,


the wage to be taken into account is
(a) The average wage earned by him in the entire service
(b) The average wage earned by him in the last 5 years
(c) The last drawn wage
(d) None of these

Financial Planning Academy 15


78 The amount of the liability of payment of gratuity is calculated at the rate of
(a) One months’ wage for each completed year of service
(b) 20 days’ wage for each completed year of service
(c) Half months’ wage for each completed year of service
(d) 15 days’ wage for each completed year of service

79 The Payment of Gratuity Act stipulates that the nomination for payment of gratuity
in case of death of an employee has to be in favour of
(a) The spouse of the employee only
(b) Any close relative of the employee
(c) Any member(s) of the family of the employee
(d) Any person as per the wish of the employee

80 The prime financial goal under retirement planning is


(a) To have assured returns
(b) To manage average income
(c) Arrange for cash flow liquidity
(d) Meeting family obligation

81 The named beneficiary in a life insurance policy is generally known as


(a) Claimant
(b) Assignee
(c) Nominee
(d) Legal heir

82 NSSO stands for


(a) National social service organization
(b) National small saving organization
(c) National sample survey organization
(d) National social security organization

83 Wealth erosion occurs on account of


(a) Increase in expenditure
(b) Accumulated debts
(c) Losses incurred
(d) Reduction in purchasing power of currency

84 Is there a maximum limit on the investment one can make in Post Office Monthly
Income scheme (POMIS)
(a) Rs.300000 in single name
(b) Rs.600000 in joint name
(c) No
(d) (a) & (b) both

Financial Planning Academy 16


85 The variation of return from expected rate of return is called
(a) Investment risk
(b) Business risk
(c) Market risk
(d) None of these

86 In the falling interest rate scenario which risk will be faced by the investors as regards
matured investments?
(a) Business risk
(b) Reinvestment risk
(c) Inflation risk
(d) None of these

87 The assets in the clients’ financial portfolio depends upon


(a) His risk appetite
(b) His wealth and income
(c) His income earning span
(d) Financial goals
(e) All of the above

88 Arithmetic mean is appropriate measure of average performance over


(a) Multiple period
(b) Single period
(c) Broken period
(d) None of these

89 Geometric mean reflects--------------- rate of growth over time.


(a) Simple
(b) Double
(c) Compound

90 Geometric mean is always ---------------------arithmetic mean


(a) Equal to
(b) Greater than
(c) Less than

91 In an inflationary period which of the following statement holds true:


(a) Nominal interest rates are lower than real interest rates
(b) Nominal interest rates are higher than real interest rates
(c) Nominal interest rates are equal to real interest rates

92 Leave salary received during the tenure of employment is


(a) Fully exempt
(b) Exempt up to a certain ceiling
(c) Fully taxable

Financial Planning Academy 17


93 Ashok has been an employee of a public sector undertaking for the past 20
years(completed) and is retiring on 1st November,2005.His company is not covered under
Payment Of Gratuity Act,1972. Ashok’s employer has agreed to pay him a gratuity of Rs
450000 on retirement. What is the tax status of this amount?
(a) The amount of gratuity payable to him cannot exceed Rs 350000
(b) The Income Tax Act will only allow a maximum gratuity of Rs 350000 to be paid
(c) The gratuity paid is exempt from Income Tax only to the maximum extent of Rs
350000

94 What will be the effect in terms of buying power on today’s terms Rs 50,000.00 after
15 years if inflation is 8% p.a.?
(a) Rs 15762.00
(b) Rs 15892.00
(c) Rs 16500.00
(d) Rs 15276.00
(HINT: FV=50000,15=N,R=8% COMP PV)

95 If the inflation rate is 4.9% and tax rate is 30%. The required rate of return to maintain
the value of an investment is:
(a) 8%
(b) 9%
(c) 7%
(d) 10%

96 Radha is an employee of a public sector undertaking(covered under the payment of


gratuity act,1972) for the past 19 years(completed) and is retiring on 31st Dec of this year.
She has completed 19 years and 2 months in job. She hopes to invest the proceeds to
provide for her retirement. Her monthly salary at the time of retirement is expected to be
Rs.21000. The amount of gratuity that she will receive will be:
(a) 230592
(b) 230192
(c) 240562
(d) 242307
(HINT: IT IS 19 COMPLETED YEARS AND ADDITIONAL MONTHS LESS THAN 6
THEREFORE YEARS WILL BE 19 ONLY)

97 Mr. Rahul ,aged 30 wants to retire at 45. He wants to maintain his present standard of
living. He spends Rs. 5,00,000 a year. He is expected to live upto 75. Inflation is assumed
at 5% and expected returns are 8%p.a. What is the real rate of return?
(a) 2.503%
(b) 1.901%
(c) 2.857%
(d) 2.568%

Financial Planning Academy 18


98 Radhika is 30 years old. She deposits 25000 at the beginning of each year in deferred
annuity scheme as a part of her retirement planning. How much will be in the account
after 25 years if it earns 9.5% compound annual interest?
(a) 2474985
(b) 2487216
(c) 2414854
(d) 2497857

99 Samir, aged 25 plans to retire at age 55. His life expectancy is 75. His current annual
expenditure is Rs 250000. He estimates no reduction in his expenses post retirement. If
interest rate is expected to be 8.5%and inflation is 5%, estimate how much will he have to
save per annum in order to achieve his target provided he does not wish to leave an
estate.
(a) Rs 129731
(b) Rs 125054
(c) Rs 117154
(d) Rs 120963
(HINT: Step-1 calculate he amount of monthly expenditure required 250000 Pv,30n,5i
compute: FV i.e.Rs 1080486.
Step-2 Calculate the amount required to be saved to enable to spend Rs 1080486 every
year for 20 years at the beginning of the year( expenses are taken at the beginning of the
year) 1080486 pmt,3.33 i(1.085/1.05-1*100) 20 n i.e. Rs 16114541
Step-3 Calculate the annual saving installment to save RS 16114541 i.e. 16114541 FV,30
n,8.5i compute pmt at beg of the year i.e Rs 129731

100 -----% of gratuity received on retirement by a central government employee is


taxable.
(a) Ten
(b) Nil
(c) Twenty five
(d) Twenty
(e) Thirty three

Financial Planning Academy 19


Question from Pg No.11 ( Mr.Rajan )

Q56. 1)Pmt = 700,000, N = 4, I= 7 , Pv= 23710479

2) Pmt= 10,00,000 , N 11, I = 7 Pv4= 74986743

Find The PV as on todays date from point 2

Fv= 74986743, N= 4, I= 7, Pv= 57206994

Now the Total Of 1) + 57206994 = 80917473.

Now we have get the future value of this amount

Pv= 80917473, I=7 , N=15 Comp FV= 223.25 lac

Q57. Life Expectancy Will ( 85 – 57) 28 year

N= 28, I= 8 , Pv= 223.25 lac , Bgn mode, Comp Pmt= 18.70 lac

Q58. Pmt= 360000, I= (1.07/1.06)-1 , Pv= 223.56 lac ,


Comp N will be Forever

Q59. Pmt= 12lac , N=5, I= 7 ,Comp Fv = 69.01lac

Pv= -223.55lac, N= 5, I= 7, Comp Fv= 313.12 lac

Total will be 382.13 lac


Q60. Age 57+ 5( Extended Contract)=62

Life Expectancy= 85 – 62 year

N=23, Pv= 382.13 lac, End mode, I= 6 , Comp Pmt= 29.30 lac

Financial Planning Academy 20


Financial Planning Academy 21
Part 2

1. A person-aged 25yrs has a yearly expense of Rs. 270000/-, which he thinks would
grow at 7% p.a. He would also like to increase his lifestyle every year by 3%. He
wishes to retire & stay with his wife at the age of 55 yrs as his father died at the age
of 65 yrs he expects the same life expectancy for himself. The returns on his
investments during his earning life would be 15%, as he believes to be aggressive in
life. The inflation post retirement would be @ 5% and he would fetch a return of 8%
in the same period, as he would be conservative in that phase of life. He would like to
know how much amount he should invest in the beginning of every year.

2. A person-aged 30yrs has a monthly expense of Rs. 35000/-, which he thinks would
grow at 6% p.a. He wishes to retire & stay with his wife at the age of 60 yrs. He
expects his life expectancy to be 75 yrs. The returns on his investments during his
earning life would be 12%, as he believes to be aggressive in life. The inflation post
retirement would be @ 5.5% and he would fetch a return of 9% in the same period, as
he would be conservative in that phase of life. He would like to know how much
amount he should invest every month.

3. A person-aged 45yrs has a monthly expense of Rs. 45000/-, which he thinks would
grow at 6.5% p.a. His post retirement expenses would be 15% less to his current
expenses. He wishes to retire & stay with his wife at the age of 58 yrs as his father
died at the age of 78 yrs he expects the same life expectancy for himself. The returns
on his investments during his earning life would be 15%, as he believes to be
aggressive in life. The inflation post retirement would be @ 4.5% and he would fetch
a return of 7.5% in the same period, as he would be conservative in that phase of life.
He would like to know how much amount he should invest every year.

4. A person-aged 33yrs has a yearly expense of Rs. 550000/-, which he thinks would
grow at 5% p.a. He would also like to increase his lifestyle every year by 2%. His
post retirement expenses would be 15% less to his current expenses. He wishes to
retire & stay with his wife at the age of 58 yrs as his father died at the age of 70 yrs he
expects the same life expectancy for himself. The returns on his investments during
his earning life would be 18%, as he believes to be aggressive in life. The inflation
post retirement would be @ 5.5% and he would fetch a return of 8.5% in the same
period, as he would be conservative in that phase of life. He would like to know how
much amount he should invest in the beginning of every month.

Financial Planning Academy 22


5. A person-aged 28yrs has a yearly expense of Rs. 350000/-, which he thinks would
grow at 7% p.a. He would also like to increase his lifestyle every year by 3%. He
wishes to retire & stay with his wife at the age of 55 yrs as his father died at the age
of 65 yrs he expects the same life expectancy for himself. The returns on his
investments during his earning life would be 10%, as he believes to be aggressive in
life. The inflation post retirement would be @ 5% and he would fetch a return of
8.5% in the same period, as he would be conservative in that phase of life. He would
like to know how much amount he should have at the time of retirement.

6. A person-aged 38yrs has a monthly expense of Rs. 25000/-, which he thinks would
grow at 5.75% p.a. He wishes to retire & stay with his wife at the age of 60 yrs as his
father died at the age of 75 yrs he expects the same life expectancy for himself. The
returns on his investments during his earning life would be 12%, as he believes to be
aggressive in life. The inflation post retirement would be @ 6% and he would fetch a
return of 9% in the same period, as he would be conservative in that phase of life. He
would like to know how much amount he should invest in the beginning of every
month.

7. A person-aged 40yrs has a yearly expense of Rs. 600000/-, which he thinks would
grow at 5% p.a. He would also like to increase his lifestyle every year by 1.5%. He
wishes to retire & stay with his wife at the age of 60 yrs as his father died at the age
of 80 yrs he expects the same life expectancy for himself. The returns on his
investments during his earning life would be 18%, as he believes to be aggressive in
life. The inflation post retirement would be @ 5% and he would fetch a return of
8.5% in the same period, as he would be conservative in that phase of life. He would
like to know how much amount he should have at the time of retirement considering
his expenses are incurred at the end of every month.

8. A person-aged 42yrs has a monthly expense of Rs. 45000/-, which he thinks would
grow at 6% p.a. His post retirement expenses would be 15% less to his current
expenses. He wishes to retire & stay with his wife at the age of 60 yrs as his father
died at the age of 75 yrs he expects the same life expectancy for himself. The returns
on his investments during his earning life would be 12%, as he believes to be
aggressive in life. The inflation post retirement would be @ 5% and he would fetch a
return of 8.5% in the same period, as he would be conservative in that phase of life.
He would like to know how much amount he should invest in the beginning of every
year.

9. A person-aged 47yrs has a monthly expense of Rs. 30000/-, which he thinks would
grow at 5.5% p.a. He wishes to retire & stay with his wife at the age of 65 yrs. He
expects his life expectancy to be 80 yrs. The returns on his investments during his
earning life would be 12%, as he believes to be aggressive in life. The inflation post
retirement would be @ 5.5% and he would fetch a return of 9% in the same period, as
he would be conservative in that phase of life. He would like to know how much
amount he should invest every month.

Financial Planning Academy 23


10. A person-aged 33yrs has a monthly expense of Rs. 50000/-, which he thinks would
grow at 7% p.a. He would also like to increase his lifestyle every year by 3%. He
wishes to retire & stay with his wife at the age of 58 yrs as his father died at the age
of 72 yrs he expects the same life expectancy for himself. The returns on his
investments during his earning life would be 15%, as he believes to be aggressive in
life. The inflation post retirement would be @ 5% and he would fetch a return of 9%
in the same period, as he would be conservative in that phase of life. He would like to
leave an estate of Rs. 15lakhs for his son, so he would like to know how much
amount he should invest in every year.

11. A person-aged 28yrs has a yearly expense of Rs. 420000/-, which he thinks would
grow at 6.5% p.a. He wishes to retire & stay with his wife at the age of 60 yrs as his
father died at the age of 78 yrs he expects the same life expectancy for himself. The
returns on his investments during his earning life would be 12.5%, as he believes to
be aggressive in life. The inflation post retirement would be @ 5.5% and he would
fetch a return of 8.5% in the same period, as he would be conservative in that phase
of life. He would like to leave an estate of Rs. 25lakhs for his son, so he would like to
know how much amount he should invest in the beginning of every month.

12. A person-aged 30yrs has a yearly expense of Rs. 550000/-, which he thinks would
grow at 7% p.a. He wishes to retire & stay with his wife at the age of 60 yrs as his
father died at the age of 75 yrs he expects the same life expectancy for himself. The
returns on his investments during his earning life would be 15%, as he believes to be
aggressive in life. The inflation post retirement would be @ 5.5% and he would fetch
a return of 9% in the same period, as he would be conservative in that phase of life.
He would like to leave an estate of Rs. 25lakhs for his son, so he would like to know
how much amount he should invest in the beginning of every year.

13. A person-aged 35yrs has a monthly expense of Rs. 45000/-, which he thinks would
grow at 6% p.a. He would also like to increase his lifestyle every year by 3%. He
wishes to retire & stay with his wife at the age of 55 yrs as his father died at the age
of 70 yrs he expects the same life expectancy for himself. The returns on his
investments during his earning life would be 15%, as he believes to be aggressive in
life. The inflation post retirement would be @ 5% and he would fetch a return of 8%
in the same period, as he would be conservative in that phase of life. He would like to
leave an estate of Rs. 25lakhs for his son, so he would like to know how much
amount he would require when he retires.

Financial Planning Academy 24


14. A person-aged 37yrs has a yearly expense of Rs. 570000/-, which he thinks would
grow at 7% p.a. He would also like to increase his lifestyle every year by 3%. He
wishes to retire & stay with his wife at the age of 58 yrs as his father died at the age
of 70 yrs he expects the same life expectancy for himself. The returns on his
investments during his earning life would be 12%, as he believes to be aggressive in
life. The inflation post retirement would be @ 5.5% and he would fetch a return of
8% in the same period, as he would be conservative in that phase of life. He would
like to know how much amount he should invest in the beginning of every year.

15. A person-aged 42yrs has a monthly expense of Rs. 60000/-, which he thinks would
grow at 7% p.a. He would also like to increase his lifestyle every year by 3%. He
wishes to retire & stay with his wife at the age of 60 yrs as his father died at the age
of 78 yrs he expects the same life expectancy for himself. The returns on his
investments during his earning life would be 13%, as he believes to be aggressive in
life. The inflation post retirement would be @ 5% and he would fetch a return of 9%
in the same period, as he would be conservative in that phase of life. He would like to
know how much amount he should invest every year.

16. A person-aged 40yrs has a yearly expense of Rs. 650000/-, which he thinks would
grow at 7% p.a. He would also like to increase his lifestyle every year by 1.5%. He
wishes to retire & stay with his wife at the age of 60 yrs as his father died at the age
of 75 yrs he expects the same life expectancy for himself. The returns on his
investments during his earning life would be 15%, as he believes to be aggressive in
life. The inflation post retirement would be @ 5.5% and he would fetch a return of
9% in the same period, as he would be conservative in that phase of life. He would
like to leave an estate of Rs. 20lakhs for his son, so he would like to know how much
amount he should invest in the beginning of every month and the corpus required at
the time of retirement.

Financial Planning Academy 25


Q-1 (83,287.04)

Annual Expenditure carried Corpus at the Beginning Investment Required to Reach Investment Required to Reach
Forward to First year of mode on the day of the Corpus at the BGN mode the Corpus at the BGN mode
Retirement. Retirement per annum per month

Present Value -270000 PMT (4,711,338.61) F V 41639932.8 F V 41,639,932.80


NPER 30 NPER 10 NPER 30 NPER 360
Inflation 10.00% Real Rate 2.85714% Investment Rate 15.00% Investment Rate 1.25%
of Return

Future Value 4,711,338.61 P V 41,639,932.80 PMT (83,287.04) PMT (5,940.23)

Q-2 (8,320.79)

Annual Expenditure carried Corpus at the Beginning Investment Required to Reach Investment Required to Reach
Forward to First year of mode on the day of the Corpus at the end mode the Corpus at the end mode per
Retirement. Retirement per annum month

Present Value -420000 PMT (2,412,266.29) F V 29,080,855.84 F V 29,080,855.84


NPER 30 NPER 15 NPER 30 NPER 360
Inflation 6.00% Real Rate 3.31754% Investment Rate 12.00% Investment Rate 1.00%
of Return

Future Value 2,412,266.29 P V 29,080,855.84 PMT (120,501.11) PMT (8,320.79)

Q-3 (469,277.39)

Annual Expenditure carried Corpus at the Beginning Investment Required to Reach Investment Required to Reach
Forward to First year of mode on the day of the Corpus at the end mode the Corpus at the end mode per
Retirement. Retirement per annum month

Present Value -459000 PMT (1,040,776.76) F V 16,120,578.07 F V 16,120,578.07


NPER 13 NPER 20 NPER 13 NPER 156
Inflation 6.50% Real Rate 2.87081% Investment Rate 15.00% Investment Rate 1.25%
of Return

Future Value 1,040,776.76 P V 16,120,578.07 PMT (469,277.39) PMT (33,899.55)

Q-4 (4,569.94)

Financial Planning Academy 26


Annual Expenditure carried Corpus at the Beginning Investment Required to Reach Investment Required to Reach
Forward to First year of mode on the day of the Corpus at the end mode the Corpus at the end mode per
Retirement. Retirement per annum month

Present Value -467500 PMT (2,537,324.76) F V 26,218,917.91 F V 26,218,917.91


NPER 25 NPER 12 NPER 25 NPER 300
Inflation 7.00% Real Rate 2.84360% Investment Rate 18.00% Investment Rate 1.50%
of Return

Future Value 2,537,324.76 P V 26,218,917.91 PMT (76,528.46) PMT (4,569.94)

Financial Planning Academy 27


Q
Q -- 95 (15,012.03)
39,766,096.41

Annual Expenditure carried Corpus at the Beginning mode on the day of Retirement
Annual Expenditure
Forward carried
to First year of Corpus at the Beginning Investment Required to Reach Investment Required to Reach
Forward to First year of
Retirement. mode on the day of the Corpus at the end mode the Corpus at the end mode per
Retirement. Retirement per annum month
Present Value -350000 PMT (4,588,497.97)
Present
NPER Value -36000027 PMT
NPER (943,727.86)
10 F V 11,377,024.92 F V 11,377,024.92
NPER
Inflation 18 NPER
10.00% Real Rate 15
3.33333% NPER 18 NPER 216
Inflation 5.50% Real Rate
of Return 3.31754% Investment Rate 12.00% Investment Rate 1.00%
of Return
Future Value 4,588,497.97 P V 39,766,096.41
Future Value 943,727.86 P V 11,377,024.92 PMT (204,073.24) PMT (15,012.03)
Q-6 (9,941.23)

Q - 10 (343,425.38)
Annual Expenditure carried Corpus at the Beginning Investment Required to Reach Investment Required to Reach
Forward to First year of mode on the day of the Corpus at the end mode the Corpus at the end mode per
Retirement. carried CorpusRetirement
Annual Expenditure at the Beginning Investmentper annum to Reach Investment Required
Required month to Reach
Forward to First year of mode on the day of the Corpus at the end mode the Corpus at the end mode per
Retirement. -300000 PMT Retirement
Present Value (1,026,335.92) F V per annum
12,755,249.16 F V month12,755,249.16
NPER 22 NPER 15 NPER 22 NPER 264
Present
InflationValue -600000
5.75% PMT
Real Rate (6,500,823.57)
2.83019% FInvestment
V Rate 73,078,522.63
12.00% FInvestment
V Rate 73,078,522.63
1.00%
NPER 25 NPER
of Return 14 NPER 25 NPER 300
Inflation 10.00% Real Rate 3.80952% Investment Rate 15.00% Investment Rate 1.25%
Future Value 1,026,335.92 ofP Return
V 12,755,249.16 PMT (137,890.73) PMT (9,941.23)
FV -1500000
Future Value 6,500,823.57 P V 73,078,522.63 PMT (343,425.38) PMT (22,530.56)
Q-7 (13,652.99)

Q - 11 (9,168.46)
Annual Expenditure carried Corpus at the Beginning Investment Required to Reach Investment Required to Reach
Forward to First year of mode on the day of the Corpus at the end mode the Corpus at the end mode per
Annual Expenditure
Retirement. carried CorpusRetirement
at the Beginning Investmentper Required
annum to Reach Investment Required
month to Reach
Forward to First year of mode on the day of the Corpus at the end mode the Corpus at the end mode per
Retirement. -600000 PMT Retirement
Present Value (2,114,187.04) F V per annum
31,522,774.10 F V month31,522,774.10
NPER 20 NPER 20 NPER 20 NPER 240
Present
InflationValue -420000
6.50% PMT
Real Rate (3,150,915.37)
3.33333% FInvestment
V Rate 46,672,736.01
18.00% FInvestment
V Rate 46,672,736.01
1.50%
NPER 32 NPER
of Return 18 NPER 32 NPER 384
Inflation 6.50% Real Rate 2.84360% Investment Rate 12.50% Investment Rate 1.04%
Future Value 2,114,187.04 of
P Return
V 31,522,774.10 PMT (214,984.73) PMT (13,652.99)
FV -2500000
Future Value 3,150,915.37 P V 46,672,736.01 PMT (122,483.70) PMT (9,168.46)
Q-8 (252,705.74)

Annual Expenditure carried Corpus at the Beginning Investment Required to Reach Investment Required to Reach
Forward to First year of mode on the day of the Corpus at the end mode the Corpus at the end mode per
Retirement. Retirement per annum month

Present Value -459000 PMT (1,310,141.67) F V 15,778,865.53 F V 15,778,865.53


NPER 18 NPER 15 NPER 18 NPER 216
Inflation 6.00% Real Rate 3.33333% Investment Rate 12.00% Investment Rate 1.00%
of Return

Future Value 1,310,141.67 P V 15,778,865.53 PMT (252,705.74) PMT (20,614.13)


Financial Planning Academy 28
Q - 12 (104,019.19)

Annual Expenditure carried Corpus at the Beginning Investment Required to Reach Investment Required to Reach
Forward to First year of mode on the day of the Corpus at the end mode the Corpus at the end mode per
Retirement. Retirement per annum month

Present Value -550000 PMT (4,186,740.27) F V 52,005,114.06 F V 52,005,114.06


NPER 30 NPER 15 NPER 30 NPER 360
Inflation 7.00% Real Rate 3.31754% Investment Rate 15.00% Investment Rate 1.25%
of Return
FV -2500000
Future Value 4,186,740.27 P V 52,005,114.06 PMT (104,019.19) PMT (7,418.89)

Q - 13 39,186,322.48

Annual Expenditure carried Corpus at the Beginning mode on the day of Retirement
Forward to First year of
Retirement.

Present Value -540000 PMT (3,026,381.81)


NPER 20 NPER 15
Inflation 9.00% Real Rate 2.85714%
of Return
FV -2500000
Future Value 3,026,381.81 P V 39,186,322.48

Q - 14 (487,916.41)

Annual Expenditure carried Corpus at the Beginning Investment Required to Reach Investment Required to Reach
Forward to First year of mode on the day of the Corpus at the end mode the Corpus at the end mode per
Retirement. Retirement per annum month

Present Value -570000 PMT (4,218,142.47) F V 44,645,612.44 F V 44,645,612.44


NPER 21 NPER 12 NPER 21 NPER 252
Inflation 10.00% Real Rate 2.36967% Investment Rate 12.00% Investment Rate 1.00%
of Return

Future Value 4,218,142.47 P V 44,645,612.44 PMT (487,916.41) PMT (39,208.42)

Financial Planning Academy 29


Q - 15 (865,634.38)

Annual Expenditure carried Corpus at the Beginning Investment Required to Reach Investment Required to Reach
Forward to First year of mode on the day of the Corpus at the end mode the Corpus at the end mode per
Retirement. Retirement per annum month

Present Value -720000 PMT (4,003,140.47) F V 53,431,401.80 F V 53,431,401.80


NPER 18 NPER 18 NPER 18 NPER 216
Inflation 10.00% Real Rate 3.80952% Investment Rate 13.00% Investment Rate 1.08%
of Return

Future Value 4,003,140.47 P V 53,431,401.80 PMT (865,634.38) PMT (62,567.74)

Q - 16 (27,232.91)

Annual Expenditure carried Corpus at the Beginning Investment Required to Reach Investment Required to Reach
Forward to First year of mode on the day of the Corpus at the end mode the Corpus at the end mode per
Retirement. Retirement per annum month

Present Value -650000 PMT (3,322,829.98) F V 41,283,872.62 F V 41,283,872.62


NPER 20 NPER 15 NPER 20 NPER 240
Inflation 8.50% Real Rate 3.31754% Investment Rate 15.00% Investment Rate 1.25%
of Return
FV -2000000
Future Value 3,322,829.98 P V 41,283,872.62 PMT (350,427.22) PMT (27,232.91)

Financial Planning Academy 30


Part 3

1) The UTI Retirement Benefit Plan offers an avenue to provide for post retirement needs
of individuals. What is the maximum permissible age for an investor to enter the scheme
without lump sum premium?
(a) 42 years
(b) 52 years
(c) 58 years
(d) 60 years

2) Pension received from a former employer is classified under the head _________.
(a) Capital Gains
(b) Gratuity
(c) Income from other sources
(d) Salaries

3) Mukesh Ambani would like to enhance her contribution and make an additional
monthly contribution of Rs. 5000. Is there a limit to the amount of voluntary contribution
Rita can make?
(a) The maximum limit is 100% of her mandatory contribution.
(b) The maximum limit is 80% of her total emoluments.
(c) The maximum limit is the value of her total emoluments per month.
(d) There is no minimum or maximum limits.

4) Shailesh has been an employee of a public sector undertaking for the past 24 years and
is retiring after one year. He is eligible for gratuity as per the provisions of the Payment
of Gratuity Act, 1972. His monthly salary at retirement is expected to be Rs. 20,000. The
amount of gratuity that he will receive on retirement will be
________.
(a) Rs. 2,88,461
(b) Rs. 3,00,000
(c) Rs. 3,40,909
(d) Rs. 3,75,000

Financial Planning Academy 31


5) Mr. Jain has been an employee of a public sector undertaking for the past 25 years and
is retiring the next year. He is eligible for gratuity as per the provisions of the Payment of
Gratuity Act, 1972. He hopes to invest the proceeds along with the PF proceeds, in order
to fund his retirement. Prior to his wedding, Mr. Jain had nominated his mother to receive
his gratuity, in the event of his death. Is his wife eligible to receive the gratuity proceeds
in the event of his death?

(a) All nominations made prior to the employee acquiring a family are invalid once
an employee has acquired a family.
(b) As long as a valid nomination is in place, the payment of gratuity will be made only
to such nominees.
(c) The payment of gratuity will always be according to the registered will, irrespective
of the nominations.
(d) The information is not sufficient for giving an answer.

6) Feroz is eligible for gratuity as per the provisions of the Payment of Gratuity Act,
1972. He has been in service only for 7 years in this company, and has taken a total leave
of 1 year and 2 months, because he needed prolonged medical attention after a major
accident. Which of the following is true about his gratuity entitlement?

(a) Feroz is eligible for gratuity, as absence on medical grounds is not added for
computing continuous service for payment of gratuity.
(b) Feroz is eligible for gratuity as long as he has worked for 240 days in the year
preceding the year in which computation of continuous service is being made.
(c) Feroz is not eligible for gratuity if has not put in continuous service of 5 years.
(d) Feroz is not eligible for gratuity if his service is interrupted by leave and does not add
up to 5 years.

7) Mr. Rao is a small-scale trader in Ball Bearing. He has a welfare scheme for his few
employees, for whom he runs a small provident fund, which is yet to be recognized. He
also contributes to this fund as an employer and matches the employee’s contribution. He
has encouraged his employees also to open PPF accounts to save tax and set aside funds
for the future. Mr. Rao has called you to take a look at what he has been doing and advise
him on the choice of products for providing retirement benefits to himself and his
employees. Can Mr. Rao open joint PPF accounts with his employees?

(a) A PPF account cannot be opened in joint names.


(b) No, only members of the same family or HUFs can hold joint accounts.
(c) Yes, it can be opened, but the proceeds cannot be made payable to either or survivor.
(d) Yes, provided the employee is the first holder.

Financial Planning Academy 32


8) How would you classify the two retirement plans, the unrecognized provident fund and
the PPF account?
(a) Both are defined benefit plans.
(b) Both are defined contribution plans.
(c) The provident fund is a defined benefit plan; the PPF is a defined contribution plan.
(d) The provident fund is a defined contribution plan; the PPF is a defined benefit plan.

9) Which is not the condition for getting supernnuation fund approved?


(a) All the benefits should be payable only in India
(b) Employee should be contributor to the fund
(c) Employer should be contributor to fund
(d) Funds have to be invested as per income tax rules1962

10) Which one of the following is a common actuarial assumption used in determining
the plan contributions needed to fund the benefits of a defined-benefit plan?
1. Investment performance
2. Employee turnover rate
3. Salary scale
4. Ratio of single versus married applicants

(a) 1 and 3 only


(b) 1,2 & 3 only
(c) 2 and 4 only
(d) 4 only

11) As an employee Kunal has come to you with his questions on supernnuation plans/
Annuity Plans to get more educated with current scenario. What is the maximum limit on
contribution to approved supernnuation fund?
(a) 2 %
(b) 10 %
(c) 27% less contribution to Provident Fund
(d) No Limit

12) Under the Employee’s Pension Scheme, 1995, the superannuation pension is decided
on the basis of the following:
(a) The actual service and the actual salary
(b) The eligible service and the eligible salary
(c) The pensionable service and the pensionable salary
(d) The pre-decided service and the pre-fixed salary

Financial Planning Academy 33


13) Mr.Bhansali estimates his opportunity cost on investments to be 12% compounded
annually. Which one of the following is the best investment opportunity for Mr.Bhansali?
(a) To receive Rs, 50000 today
(b) To receive Rs. 250000 at the end of 14 years.
(c) To receive Rs. 40000 at the end of four years and Rs. 120000 eight years later (at the
end of the 12th year).
(d) To receive Rs. 5000 at the beginning of each six-month period for nine years
compounded semiannually.

14) Sachin has been investing Rs. 4000 into a mutual fund at the end of each month for
the last 10 years and has been earning a compound return of 12%, consisting entirely of
capital appreciation. Does Sachin have enough money, after selling his investments to
purchase his dream home for Rs.850000?
(a) No because he has Rs. 763595 after sale of investment and after paying long term
capital gains tax
(b) Yes because he has Rs. 920154 after sale of investment
(c) No because he has Rs. 844108 after sale of investment and after paying long term
capital gains tax
(d) No because he has Rs. 724622 after sale of investment and after paying long term
capital gains tax

15) Karan, age 45, can refinance Rs. 114000 at a 20-year rate for 8% and will incur
closing cost of 4% of the mortgage amount to be financed in the new mortgage balance.
What will be his new EMI on the mortgage under the circumstances to achieve his
objective of no debt at retirement (age 60)?
(a) Rs. 986.29
(b) Rs. 1065
(c) Rs. 1000
(d) Rs. 1133

16) Which of the following statement is true?


a) Under the Chilean Model, pension is paid to the widow of the worker on his death.
b) Under the Chilean Model, pension is paid to the worker on retirement.
c) Under the Chilean Model, pension is payable to the dependant children of the deceased
worker.
d) All the above.

Financial Planning Academy 34


17) An auditor of an Exempted Provident can be:
(a) A member of FPSB India
(b) A practising Chartered Accountant
(c) An employee of the Employee Provident Fund Organisation
(d) The auditor of the company, which is having the exempted provident fund

18) Deepa Mehta aged 30, is interested in planning for retirement. She saves Rs. 15000
per year (at the year end) in a bank fixed deposit earning 8% p.a. compounded annually
until she retires at age 60. Her life expectancy is 80 years. What will be her corpus on the
date of retirement? What is the fixed annual amount she can withdraw at the beginning of
each year until age 80, in case she wishes to exhaust her corpus completely?
(a) 1448974, 137498
(b) 1624894, 150458
(c) 1699250, 160252
(d) 1491655, 91613

Solution: - (c) Rs 1699250, Rs 160250


Retirement corpus after 30 years, with an annual savings @ 8 % will be Rs 1699250
Post retirement, annuity due for 22 years will Rs 160252

19) Ms. Rekha is 45 years old and plans to retire at 50. Her life expectancy is 70 years.
Ms. Sushma her Financial Planner, estimates that her client will require Rs.45000 in the
first month after retirement BGN . Inflation rate is 4% p.a. and the rate of return is 6%
p.a. What will be the savings per year required in order to meet this?
(a) 1245879
(b) 1478951
(c) 1589420
(d) 1689745

Rate of return after inflation adjustment is 1.923077 (((1.06/1.04)-1)*100)


Retirement corpus at age 50 will be Rs 89,59,710.45 ( They assumed End Mode.)
Annual savings @6% for the retirement corpus will be Rs 15,89,420

20) Mr. Rao retired from ITI Ltd after completing service of 30 years . His salary at the
time of retirement was Rs10, 500 per month while the average salary drawn for the
preceding 10 months worked out to Rs 9, 800 per month. The actual amount of gratuity
received by him at the time of retirement was Rs 3,25,000. Calculate the amount of
gratuity exempt from tax assuming that the provisions of Payment of Gratuity Act,
1972,cover him?
(a) Rs. 1,47,000
(b) Rs. 175,673
(c) Rs. 2,94,000
(d) Rs. 1,81,730

Financial Planning Academy 35


21) Mr. Sahai has just retired from Govt. service with a lump sum of Rs 2600000 as
retirement benefits in total. Currently he is 59 and life expectancy for him is 75 years. He
intends to take a world trip after 4 years, which would entail an amount of Rs 5 lakhs at
current prices and wants to buy a new car of Rs 3 lakhs immediately.
Calculate what amount will be available to him for post retirement living expenses in the
beginning of every month, considering inflation @ 4.5 % and rate of return is 8 % p.a?

(a) Rs 11,889
(b) Rs 12,195
(c) Rs 12,090
(d) Rs 12,486

Net Retirement corpus available is Rs 18,00,000 after deducting Rs 8lakhs


(5 lakhs + 3 lakhs)
Inflation adjusted rate of return is 3.35 %
Monthly amount, at the beginning for next 16 years will Rs 12,090

22) Jaishank is a young professional ageing 27 years, who has started investing in a ULIP
of a Insurance company. His annual contribution is Rs 60,000 in the beginning of the
year. He has opted for balanced fund looking at the bear phase of the market. He is
optimistic and believes that the market will rise and like wise interested in moving to
growth fund, say after 2 years and is considering to a protector fund option 5 years before
retirement, which as per his company policy is 58 years. Considering rate of return for
growth fund be 12 %, balanced fund be 8 % and protector fund be 6 %, what will be the
accumulated value of the ULIP if initially, investible amount of the contribution is 70 %,
increasing by 10 % for subsequent years?

(a) Rs 1,71,06,852
(b) Rs 76,20,539
(c) Rs 1,69,56,625
(d) Rs 1,29,10,277

Investible amount for first, second & third years would be Rs 42,000, Rs 48,000 & Rs
54,000 respectively.
Rate of return for first two years would be 8 %, next 23 years would be 12 % and for last
five years would be 6 %
Total accumulated amount at the retirement would be Rs 1,29,10,277

Financial Planning Academy 36


Q 22 .This Question will be solved in 3 parts, which are as follow

Part 1
1. PV = - 42000, N = 1 , I = 8 , Comp Fv = 45360
2. PV = -45360 , Nper = 1 , PMT = -48000 , I= (1.08)^2 = 1.1664, BGN ,
COMP Fv = 100828.80

Part 2
1. PMT = -54000, PV = -100828.80 , n = 1 , I = 12 % , BGN , Comp FV = 173408.26

2. PMT = -60000 , PV= -173408.26 , I =12 % , N = 23 , BGN , Comp FV = 9379403.37

Part 3
PMT = -60000 , PV = -9379403.37 , N = 5 , I = 6 % , BGN, Comp FV = 1,29,10,276.6

Financial Planning Academy 37


Part 4

1. Ms. M is 40 yrs old to retire at 65. Life expectancy is 75 yrs. She will require 15000 in
1st month after retirement. Inflation 4% p.a., rate of return 7% . What is the corpus
required to meet the expenses after retirement. Will the corpus be enough to fund her
retirement if she saves upto Rs. 30,000 pa (at the end of the year)

(a) 1478597 , Yes


(b) 1687498, No
(c) 1984571, No
(d) 1562027 , Y

2. Nirav wants to retire at 45 and he wants to maintain his present standard of living. He
spends 325000 a year. He is expected to live upto 85. Inflation 4% expected return 7%
pa . How can he achieve this? He is at present 30 yr. What is the nest egg required at age
45 and what amount shall he save every year to meet this plan? His present investment is
Rs.10,00,000.

(a) Nest egg and saving required will be 25100065 and 541093 resp.
(b) Nest egg and saving required will be 12773065 and 438300 resp.
(c) Nest egg and saving required will be 13785155 and 439780 resp.
(d) Nest egg and saving required will be 14773065 and 740530 resp.

3. Mira aged 30 saves Rs. 15000 per year (at the end) in Bank FD earning 8.25% p.a.
comp annually until she retires at 58. Life expectancy is 80 yrs. Corpus on the date of
retirement ? What is the fixed amount she can withdraw at the beginning of each yr until
80 in case she wishes to exhaust her corpus completely. Inflation rate 5% pa.

(a) 1424894, 89458


(b) 1348974 , 87498
(c) 1491655, 137767
(d) 1491655, 91613

4. Rekha 20 yrs retired at 45. Life expectancy 70. She requires 55000 in the first month of
her retirement. Inflation rate 4% pa rate of return 6% pa. What will be the saving per year
required in order to meet this.
(a) 232584
(b) 235789
(c) 236478
(d) 238615

Financial Planning Academy 38


5. Mr. Prakash is 35 yrs old to retire at 60. Life expectancy is 80 yrs. He will require
25000 in 1st month after retirement. Inflation 5% p.a. , rate of return 8% . What is the
corpus required to meet the expenses after retirement. Will the corpus be enough to fund
her retirement if he saves upto Rs. 48,000 pa (at the end of the year)

(a) 4129874, Y
(b) 4784962, N
(c) 4566431, N
(d) 4378154, Y

6. Kalpesh wants to accumulate Rs.50 Lakh by retirement. He is 30 yrs and wants to


retire at 55 yrs. Interest rate 9 % inflation 5% compounded yearly to be done on annuity
certain basis. After 10 yrs of his saving 45000pa. Kalpesh now realises he can now
earn12 % pa on fresh investment and also maintain a saving amount of 72000 pa for the
rest of the working life. Will he be able to accumulate the amount required? What will his
corpus be

(a) Y , 64.26 Lakh


(b) N, 48.75 Lakh
(c) Y, 52.5 Lakh
(d) N, 49.50Lakh

7. Krishna aged 30 saves Rs. 10000 per year (at the end) in Bank FD earning 8% p.a.
comp annually until he retires at 60. Life expectancy is 80 yrs. Corpus on the date of
retirement ? What is the fixed amount she can withdraw at the beginning of each yr until
80. Presuming he wishes to leave his heirs an estate of Rs. 200,000. Inflation rate is
constant at 6% pa.

(a) 1105694, 61375


(b) 1132832, 12787
(c) 1054875, 108423
(d) 1132832, 59040

8. Manoj 30 yrs employee earning salary of Rs. 300000. He started saving 10 % of his
salary at the end of the year in a saving plan which yields 6% interest pa. His salary
increases by 5% pa. If Manoj intends to prepone his retirement to the age of 55 yrs and
needs to have the same amount of accumulated saving as at the age of 60 yrs. What
percentage of his salary should he start saving to achieve his goal.

(a) 14.04%
(b) 13.56%
(c) 15.70%
(d) 16.70%

Financial Planning Academy 39


9. Ramesh retired from PTC completed service of 29 yrs 9 month. His salary at
retirement was Rs. 10,500 per month. While average salary of preceding 10 months
worked out to Rs. 9800 p.m. The actual amount of gratuity received by him at retirement
was Rs.325000. Calculate the amount of gratuity exempt from tax assuming he is
covered by provision of Payment of gratuity Act 1972.

(a) 294000
(b) 181730
(c) 315000
(d) 175673

10. Shinde invested Rs. 72000 at the rate of interest of 5%. After 7 yrs the ROI is 5%
compounded half yearly. After 3 yrs ROI is 6% compounded quarterly. What will he get
after 15 years.

(a) 140000
(b) 155000
(c) 148251
(d) 158242

11. Mukesh retires from service received Rs 10 Lakh in total tax free retirement benefit.
He will receive Rs. 10,000 p.m. as pension but his proposed expenditures are 200% of
what he receives. He is a senior citizen and wants to invest his lump sum amount in a
scheme which provides safety of principle, regular and max. Interest. He is open to invest
in MF (MIP ) or like fund but only as second option. Assuming return on debt and MF
(MIP) is around 8% pa. What is the maximum amount he should put in Post Office
Senior citizen Saving Scheme so that his tax liability remains NIL without requiring to
invest under section 80C. If he invests rest of his fund in MF (MIP) by what % he needs
to reduce his proposed monthly expenditure to meet the deficit in interest earning.

(a) 722000, 13.65%


(b) 1000000, 13.65
(c) 1000000,10%
(d) 722000,10%

13. Ram aged 35 saves Rs. 30000 per year (at the end) in Bank FD earning 7% p.a. comp
annually until he retires at 60. Life expectancy is 70 yrs. Corpus on the date of retirement
What is the fixed amount she can withdraw at the beginning of each yr until 70?
Presuming he wishes to leave his heirs an estate of Rs. 150,000. Inflation rate is constant
at 5% pa.

(a) 1897471, 242337


(b) 2147854, 214598
(c) 1745897, 192712
(d) 1945875, 199145

Financial Planning Academy 40


Solution For Part 4 of Retirement Planning Module :

1. First find the PV of cash flow series (i.e 15000pm from year 65 to 75 in total 10
years of post Retirement.

Pmt = 15000, N= 10, I = {(1.07/1.04)-1} this is nothing but Inflation adjusted return,
End Mode Comp Pv = 1562027

Second Part is the Saving of 30000 pa.


Pmt= -30000, N= 25, I= 7, Comp FV = 18,97,471

2. Step 1. to find FV of current spending @325000 p.a after 15 year inflation @ 4%

Pv = -325000, N= 15, I = 4%, Comp Fv = 585307

Step 2. To Find PV of cash flow series (i.e 585307 pa from year 45 to 85 in total 40
years of post Retirement.)

Pmt = -585307, N= 40, I = {(1.07/1.04)-1} this is nothing but Inflation adjusted


return, End mode, Comp PV = 13785170.

Step 3. Find FV of Current Invt.

Pv = -10,00,000, N = 15 , I= 7%, Comp Fv = 27,59,032

Step 4. To find Saving require to achieve his goal.

The amount require would be difference between 13785170 – 2759032 = 11026138

The saving per year would be.

Fv = 11026138, N= 15, I= 7, Comp Pmt = 438780.

3. FV of Current Saving.

Pmt = -15000 , N= 28, I= 8.25% , Comp Fv= 1491655.

Amount that can be withdrawn will be :

Pv = 1491655, N= 22, I= {(1.0825/1.05)-1} this is nothing but Inflation adjusted


return, BGN mode , Comp PMt = 91613.

Financial Planning Academy 41


4. Pmt = -55000, N= 25year*12mth, I= (1.06/1.04)-1, Comp PV= 13091516

Saving Part :

Fv = 13091516, N= 25, I= 6, Comp Pmt= 238615

5. Pmt = -25000, N= 20*12, I= (1.08/1.05)-1, Comp Pv = 4566431.

Pmt = -48000, N = 25, I= 8, Comp Fv= 3509085.

6. First Part :

Pmt= -45000, N= 10, I= 9, Comp Fv = 683682

Second Part :

Pmt =-72000, I= 12, N= 15, Pv = -683682, Comp Fv= 64.26 Lakh.

7. First Part :

Pmt= -10000, N=30, I=8, Comp Fv= 1132832

Second PaRT, find the PV of the estate which he leave behind

Fv =200000, N= 20, I= (1.08/1.06)-1, Comp Pv = 137617

For Withdrawal part , we have to Get difference amount Fv of part 1 & Pv of part 2
Which comes to 995215.This the amount available for withdrawal.

Third Part :

Pv = -995215, I= (1.08/1.06 )-1, N= 20, BGN Mode, Comp Pmt= 59087.

Financial Planning Academy 42


8.The Q8 Is very Imp from Examination Point of View.

These question can be Solved with the help os a Formula, which is

A{(1+r)^n- (1+g)^n / r - g}

First Part :

Saving Element is 10% of Annual Earning


Which is 300000*10%= 30000

30000{(1.06)^30 – (1.05)^30 / .06 - .05}

= 4264650.

If he Prepones his retirement to 55 years then Future Service period


55-30= 25 years

But he wants the same accumulated amount of Rs.4264650 as calculated above at


his early retirement age of 55 years.

Let ‘ X’ be the amount of saving in the first , increasing thereafter by 5% every year

Then 4264650= X { (1.06)^25 – (1.05)^25 / .06 - .05}

X= 4264650/90.552

X= 47096.14

This amount of Rs. 47096.14 will have to be saved during the first year, Since the
salary will increase by 5% every year , this saving amount will also increase.

Saving as % of the current salary

(47096 / 300000)*100

15.70%

9. For the person covered under POGA 1972, Gratuity will be calculated as

10500*15/26*30 = 181730

Therefore from tax exemption is Rs.181730

Financial Planning Academy 43


10. Pv = -72000, N= 7, I= 5, Comp Fv = 101311

Pv= -101311, N= 3*2, I= 5/2, Comp Fv= 117489.97

Pv= -117489.97, N= 5*4, I= 6/4, Comp Fv= 158241.95

11. Senior Citizen Scheme @9%.

For the maximum amount to be invested is


185000 – 120000 = 65000.

Therefore to earn 65000 pa @ 9% he need to invest 722000 in SSS.

13.As per solution No.7

Financial Planning Academy 44


Part 5

1. Retirement planning requires meeting the diversified needs of today’s retiree and
must be approached from several perspectives. What is the most essential retirement
objective?
a. Achieving financial freedom
b. Maintaining pre-retirement standard of living
c. Retiring early
d. Improving retirement lifestyle

2. Which of the following are characteristics of a well-structured non-approved plan?


i. The plan promises fully secured benefits
ii. The plan enables deferral of taxation until the benefit is received
iii. The plan benefits key employees
a. i, ii
b. i, iii
c. ii, iii
d. i, ii, iii

3. Which of the following statements concerning retirement planning for an individual


are correct?
i. The individual’s disposable income can fall by the amount being saved for
retirement, thus increasing the individual’s current standard of living
ii. A replacement ratio of 70 to 90 percent of the average salary for the last 3
years before retirement is typically used.
iii. Retirement needs would have been overestimated if actual retirement occurs
after the planned retirement date
a. i, ii
b. i, iii
c. ii, iii
d. i, ii, iii

4. The following ways to help clients overcome inadequate retirement resources are
correct EXCEPT
a. Relocating to a less expensive home
b. Sub-letting vacant rooms in their residence
c. Continue to work after retirement age
d. Increasing the assumed EPF rate of return

Financial Planning Academy 45


5. To determine the amount of life insurance coverage that provides a surviving spouse
with a monthly income of a given purchasing power, it is necessary to consider which
of the following?
i. Life expectancy
ii. Interest rate
iii. Inflation rate
iv. Taxation
a. i, ii, iii
b. i, ii, iv
c. ii, iii, iv
d. i, ii, iii, iv

6. Which of the following is not needed to calculate the client’s target retirement fund?
a. Average annual earnings in the last 3 years before retirement
b. Estimated pension benefits during retirement
c. Costs of self-maintenance
d. Capitalization rate

7. As a client approaches retirement, he becomes less growth-oriented in his investment


approach because of the following reasons EXCEPT:
Possible Answers (four):
a. Less time to recover investment losses
b. Sufficiency of income to maintain pre-retirement standard of living
c. A decline in his risk tolerance level
d. Additional personal financial responsibilities

8. The following are reasons why an employer should adopt a defined-benefit plan
EXCEPT
a. Long-service employees will benefit if past service is taken into account
b. A past-service provision can maximise investment returns of the plan’s
assets
c. Plan assets are not allocated to participant accounts
d. Providing a standard of living in retirement that is comparable to that enjoyed
prior to retirement

Financial Planning Academy 46


9 An individual is divorced and later remarries. His former wife is his EPF beneficiary
but in his current Will, he leaves everything to his present wife. Who will get his EPF
proceeds should he die now?
Possible Answers (four):
a. Former wife
b. Present wife
c. Each gets half
d. The court has to decide

10 Chavan and his wife could never agree on who should be the guardian of their minor
children. Chavan wants his brother, while she wants her best friend to be the guardian.
Accordingly, they did not name any guardian in their respective Wills. If both of them
pass away, who becomes the guardian?
Possible Answers (four):
a. Whichever party who can prove that he is the rightful guardian
b. Chavan’s brother, because he is a blood relative
c. Whoever is decided by the executor
d. Whatever is decided by the court

11 Lokesh’s father has given him general power of attorney. What does this mean?
Possible Answers (four):
a. He has given Larry the right to appoint himself as the sole beneficiary of his
estate
b. He has given Lokesh the immediate right to make decisions in all matters and
take action on his behalf
c. He has given Lokesh the right to make decisions in all matters and take action
on his behalf should he become incompetent.
d. He has disinherited Lokesh, but Lokesh has the right to decide who will inherit his
father’s estate.

12 In GM Rao’s Will, he uses the term “Children”, without naming his two children, in
bequeathing equally his estate worth 900,000. Furthermore, he does not mention
any possible addition to his family. Later, he has a third child. What would this child
be entitled to if Tan passes away?
Possible Answers (four):
a. Nothing, as he was not born when GM Rao made the Will
b. A sum to be decided by the judge
c. A sum to be decided by GM Rao’s executor
d. An equal share in GM Rao’s estate of 900,000

Financial Planning Academy 47

Você também pode gostar